Está en la página 1de 14

Todo sobre normas

Con la colaboración de Carlos Antunes, David Rosa, Ana Emilia de Orellana, Itatı́ Zocola
Análisis Matricial - LMA

1. Definiciones
Definición 1. Sea d : X×X → R, con X ̸= ∅ decimos que d(·, ·) es una distancia
si para todo x, y, z ∈ X se satisfacen las siguientes propiedades:

1. d(x, y) = d(y, x).


2. d(x, y) ≥ 0 y d(x, y) = 0 si y solo si x = y.
3. d(x, z) ≤ d(x, y) + d(y, z).

Definición 2. Sea ∥ · ∥ : V → R, con V un espacio vectorial, decimos que ∥ · ∥ es


una norma si para todo x, y ∈ V, c ∈ R se satisfacen las siguientes propiedades:
1. ∥cx∥ = |c|∥x∥.
2. ∥x∥ ≥ 0 y ∥x∥ = 0 si y solo si x = 0.

3. ∥x + y∥ ≤ ∥x∥ + ∥y∥.
Definición 3. Sea ⟨·, ·⟩ : V×V → R, V un e.v., decimos que ⟨·, ·⟩ es un producto
escalar si para todo x, y, z ∈ V, c ∈ R se satisfacen las siguientes propiedades:
1. ⟨x, y⟩ = ⟨y, x⟩.

2. ⟨cx, y⟩ = c⟨x, y⟩.


3. ⟨x + y, z⟩ = ⟨x, z⟩ + ⟨y, z⟩.
4. ⟨x, x⟩ ≥ 0 y ⟨x, x⟩ = 0 si y solo si x = 0.

1
1.1. Relación norma - producto interno
Todo producto escalar ⟨·, ·⟩ determina una norma definida como
1
∥x∥ = ⟨x, x⟩ 2 .
Veamos que es una norma. Sean x, y ∈ Rn , c ∈ R:
1 1 1
1. ∥cx∥ = ⟨cx, cx⟩ 2 = c2 ⟨x, x⟩ 2 = |c|⟨x, x⟩ 2 = |c|∥x∥.
1
2. ∥x∥ = ⟨x, x⟩ 2 ≥ 0.
3. ∥x + y∥2 = ⟨x + y, x + y⟩ = ⟨x, x⟩ + ⟨y, y⟩ + 2⟨x, y⟩ ≤ ∥x∥2 + ∥y∥2 +
2∥x∥∥y∥ = (∥x∥ + ∥y∥)2 .
No toda norma ∥ · ∥ proviene de un producto interno.
Consideramos la norma infinito, con ∥x∥∞ = max|xi |, se puede verificar que
cumple con las propiedades con las que se define una norma, sin embargo no
satisface la tercer propiedad de producto escalar agregar cuentitas.
Sin embargo, es cierto que esto sucede si y sólo si ∥ · ∥ satisface la identidad
del paralelogramo:
1 2 2 2 2
(∥x + y∥ + ∥x − y∥ ) = ∥x∥ + ∥y∥ .
2
Sobre la prueba de este resultado: La primer implicación no es difı́cil de
demostrar, bsata usar las propiedades de producto interno, la recı́proca está
detallada en el Anexo 2.3
Veamos a continuación que la inclusión es estricta, es decir que poseer una
distancia no implica obtener una norma y, del mismo modo, tener una norma
no nos garantiza un producto interno asociado.

1.2. Relación norma - distancia


Toda norma ∥ · ∥ determina una distancia definida como

d(x, y) = ∥y − x∥.
Veamos que es una distancia. Sean x, y, z ∈ Rn :

1. d(x, y) = ∥y − x∥ = | − 1|∥y − x∥ = ∥ − 1(y − x)∥ = ∥x − y∥ = d(y, x).


2. d(x, y) = ∥y − x∥ ≥ 0.
3. d(x, z) = ∥z−x∥ = ∥z−x+y−y∥ ≤ ∥z−y∥+∥y−x∥ = d(y, z)+d(x, y).

No toda distancia es (inducida por alguna norma. Consideramos la


1, x ̸= y
distancia discreta d(x, y) = Puede corroborarse que efectivamente
0, x = y.

2
es una distancia, pero lo que de verdad nos interesa es que no es norma ya que
no cumple la primer propiedad. En efecto, sea λ ∈ R:

∥λx∥ = d(λx, 0) = 1,
∥x∥ = d(x, 0) = 1

Por lo tanto |λ|∥x∥ = |λ| ≠ ∥λx∥, tomando λ ̸= 0.


Sin embargo se tiene esta relación: una distancia d es inducida por alguna
norma si y sólo si para todo x e y satisface

1. d(x + a, y + a) = d(x, y) para todo a ∈ Rn (invarianza traslacional)


2. d(λx, λy) = |λ|d(x, y) (homogeneidad)

Primero veamos que si d satisface ambas propiedades, entonces ∥x∥ := d(x, 0)


es una norma, y d es inducida por esta.

∥x∥ = d(x, 0) = 0 ⇐⇒ x = 0 pues d es distancia.


∥λx∥ = d(λx, 0) = |λ|d(x, 0) = |λ|∥x∥ por la homogeneidad de d.
∥x + y∥ = d(x + y, 0) ≤ d(x, 0) + d(y, 0) = ∥x∥ + ∥y∥ por la propiedad
triangular de d.

Luego ∥ · ∥ es una norma. Observemos que d(x, y) = d(x − y, 0) = ∥x − y∥


para cualesquiera x, y, por lo tanto, d es inducida por ∥ · ∥ = d(·, 0).

En el sentido inverso, si d es una distancia inducida por una norma ∥ · ∥ es


trivial que d satisface (1) y (2).

1. d(x + a, y + a) = ∥x + a − (y + a)∥ = ∥x − y∥ = d(x, y).


2. d(λx, λy) = ∥λx − λy∥ = |λ|∥x − y∥ = |λ|d(x, y).

2. Normas matriciales para matrices cuadradas


en Rn×n
2
Toda norma de Rn puede “medir” una matriz como si fuese un vector
columna de n2 × 1. Sin embargo, el espacio de las matrices tiene un producto
y por lo tanto hay que relacionar la norma de un producto de matrices con la
norma de cada una de ellas.
Definición 4. Una norma es norma matricial y se denota |||·||| si para toda
A ∈ Mn×n , c ∈ R se satisfacen las siguientes propiedades:

1. |||A||| ≥ 0 y |||A||| = 0 si y solo si A = 0.

3
2. |||cA||| = |c||||A|||.
3. |||A + B||| ≤ |||A||| + |||B|||.
4. |||AB||| ≤ |||A||||||B||| .

2.1. Normas matriciales inducidas


Dada una norma ∥ · ∥ de Rn definimos una norma inducida en las matrices
de n × n de la siguiente forma:

∥Ax∥
|||A||| = máx ∥Ax∥ = máx .
∥x∥=1 x̸=0 ∥x∥

Veamos que las normas inducidsas son normas matriciales. Sean A, B ∈


Mn×n , x ∈ Rn , c ∈ R:

1. Como ∥Ax∥ ≥ 0 para todo x entonces en particular lo es máx∥x∥=1 ∥Ax∥.


2. |||cA||| = máx∥x∥=1 ∥cAx∥ = máx∥x∥=1 |c|∥Ax∥ = |c| máx∥x∥=1 ∥Ax∥
3. |||A + B||| = máx∥x∥=1 ∥(A+B)x∥ = máx∥x∥=1 ∥Ax+Bx∥ ≤ máx∥x∥=1 (∥Ax∥+
∥Bx∥) ≤ máx∥x∥=1 ∥Ax∥ + máx∥x∥=1 ∥Bx∥ = |||A||| + |||B|||

4. |||AB||| = máx∥x∥=1 ∥ABx∥ ≤ máx∥x=1∥ |||A|||∥Bx∥ = |||A||| máx∥x∥=1 ∥Bx∥ =


|||A||||||B|||.

Es claro que el conjunto de normas sobre Mn×n es mayor que el conjunto de


normas matriciales que, a su vez, es mayor al de normas matriciales induidas.
Veamos que estas inclusiones son estrictas.

La norma l∞ = máx 1≤j≤n |aij | es una norma pero no es norma matricial.


1≤i≤n
Sea
 
1 1
A=
1 1
∥A2 ∥∞ = 1
 
2 2
A=
2 2
∥A2 ∥∞ = 2 y deberı́a ser ∥A2 ∥ ≤ ∥A∥∥A∥ para satisfacer la cuarta condición
de la definición de norma.

La norma matricial |||A||| = máx{|||A|||1 , |||A|||∞ } no es norma inducida.


Verifiquemos satisface las 4 condiciones necesarias para ser norma matricial.

4
1. |||A||| ≥ 0 ya que tanto |||A|||1 como |||A|||∞ son no negativas por tratarse
de normas, además |||A||| = 0 sı́ y solo sı́ |||A|||1 = 0 y |||A|||∞ , por lo que A
debe ser 0 .
2. |||cA||| = máx{|||cA|||1 , |||cA|||∞ } = máx{|c||||A|||1 , |c||||A|||∞ } = |c| máx{|||A|||1 , |||A|||∞ } =
|c||||A|||.
3.

|||A + B||| = máx{|||A + B|||1 , |||A + B|||∞ }


≤ máx{|||A|||1 + |||B|||1 , |||A|||∞ + |||B|||∞ }
≤ máx{|||A|||1 , |||A|||∞ } + máx{|||B|||1 , |||B|||∞ }
=|||A||| + |||B|||.

4.

|||AB||| = máx{|||AB|||1 , |||AB|||∞ }


≤ máx{|||A|||1 |||B|||1 , |||A|||∞ |||B|||∞ }
≤ máx{|||A|||1 , |||A|||∞ } máx{|||B|||1 , |||B|||∞ }
=|||A||||||B|||.

Sin embargo, esta norma matricial no es inducida ya que |||I2×2 ||| = máx{|||A|||1 , |||A|||∞ } =
máx{2, 1} = 2, y veremos a continuación que las normas inducidas satisfacen
que |||I||| = 1.
Conclusón de esta sección las normas inducidas en las matrices de n × n son
menos que las normas matriciales en las matrices de n que son menos que las
normas en las matrices de n × n.

2.2. Algunas cosas interesantes


Sea |||A||| = máx∥x∥=1 ∥Ax∥ la norma inducida por ∥ · ∥, ésta cumple las
siguientes propiedades:

a) |||I||| = máx∥x∥=1 ∥Ix∥ = máx∥x∥=1 ∥x∥ = 1


b) ∥Ay∥ ≤ |||A|||∥y∥, para todo A ∈ Mn×n , y ∈ R. Donde la igualdad es
y
válida si y = 0. Si y ̸= 0 basta tomar el vector ∥y∥ , tenemos entonces
Ay ∥Ay∥
|||A||| = máx∥x∥=1 ∥Ax∥ ≥ ∥ ∥y∥ ∥= ∥y∥ .

Las normas que cumplen |||I||| = 1 se llaman unitarias. Sabemos entonces


que toda norma inducida es unitaria, el recı́proco, sin embargo, no es cierto, la
norma l∞ es unitaria pero no es norma matricial.
P  12
1 n
La norma de Frobenius, definida como |||A|||F = tr(AAT ) 2 = i,j=1 |aij |2
es una norma heredada y es norma matricial:

5
P  12 P  12
n Pn 2 n Pn  Pn
|||AB|||F = i,j=1 | k=1 aik bkj | ≤ i,j=1 k=1 |aik |2 m=1 |bmj |2
P  12 P  12
n n
= i,k=1 |a2ij | m,j=1 |bij |2
= |||A|||F |||B|||F

Como ∥I∥2 = n entonces por el contrarecı́proco de la primera propiedad,
la norma de Frobenius no es una norma inducida.

Las normas l1 , l2 y l∞ inducen sobre Mn×n las normas |||A|||1 , |||A|||2 y |||A|||∞
respectivamente, para las cuales se puede probar las siguientes igualdades:
Pn
1. |||A|||1 = máxj i |aij | = mayor norma l1 de las columnas de A.
2. |||A|||2 = σ1 (A).
Pn
3. |||A|||∞ = máxi j |aij | = mayor norma l1 de las filas de A.

Probemos las tres igualdades:

2. Sean a1 , . . . , an las columnas de A, y x ∈ Rn de norma unitaria. Por un


lado se tiene que ∥Ax∥1 está acotado para todo x de norma unitaria, y por lo
tanto también lo está |||A|||1 = máx∥x∥1 =1 ∥Ax∥1 .

∥Ax∥1 = ∥x
Pn1 a1 + . . . + xn an ∥1
≤ |x |∥a ∥
Pnj j j 1
≤ j |x j |(máxk ∥ak ∥1 ) = ∥x∥1 (máxk ∥ak ∥1 ) = máxk ∥ak ∥1 .

Entonces máxk ∥ak ∥1 es una cota superior para |||A|||1 . Tomando ak ∗ la columna
con norma l1 más grande de A, se tiene que ∥Aek∗ ∥1 = ∥ak∗ ∥1 = máxk ∥ak ∥1 .
Como la igualdad se alcanza, (1) queda demostrado.

1. Primero observemos que la norma euclı́dea de Rn es ortogonalmente


invariante: ∥Qx∥2 = ∥x|2 para toda Q ortogonal. Sea A = U ΣV T una SVD
de A. Dado que U y V son ortogonales, y en particular V T es inversible se tiene

máx∥x∥2 =1 ∥Ax∥2 = máx∥x∥2 =1 ∥U ΣV T x∥2 = máx∥x∥2 =1 ∥ΣV T x∥2


= máx∥y∥2 =1 ∥Σy∥2
= máx∥y∥2 =1 ∥(σ1 y1 , . . . , σn yn )∥2
≤ máx∥y∥2 =1 ∥σ1 y∥2 = σ1 máx∥y∥2 =1 ∥y∥2 = σ1 .

Hemos probado que σ1 (A) es una cota superior para |||A|||2 . Como A =
σ1 u1 v1 T +. . .+σn un vn T y {u1 , . . . , un } y {v1 , . . . , vn } son conjuntos ortonormales,
se tiene que ∥Av1 ∥2 = ∥U ΣV T v1 ∥2 = ∥σ1 u1 ∥2 = σ1 . Esto es, la igualdad se
alcanza para el vector v1 . Luego |||A|||2 = σ1 (A).

6
Pn Pn
3. Probemos que valen |||A|||∞ ≤ máxi j |aij | y máxi j |aij | ≤ |||A|||∞ .
Para la primera desigualdad, observemos
Pn que tomando v = (1, . . . , 1) se tiene
∥Av∥∞ /∥v∥∞ = ∥Av∥∞ = máxi j |aij |. Por definición |||A|||∞ debe ser mayor
o igual que ∥Av∥∞ /∥v∥∞ y la primera desigualdad vale.
Para la segunda, veamos todos los cocientes están acotados por arriba. Esto
es, para todo x = con ∥x∥∞ = máxj |xj | = 1 se tiene
n
X n
X
∥Ax∥∞ = máx |aij ||xj | ≤ máx |aij |.
i i
j j

Luego vale la igualdad, que era lo que se querı́a demostrar.

2.3. Radio espectral


Sea A ∈ Mn×n , el radio espectral de A, denotado ρ(A), se define como el
máximo de los módulos de los autovalores de A, esto es:

ρ(A) = máx {|λi | : λ1 , · · · , λn autovalores deA}.


1≤i≤n

Teorema: Para toda A ∈ Rn×n y ϵ > 0 existe una norma matricial |||·||| tal
que ρ(A) ≤ |||A||| < ρ(A) + ϵ.

En la prueba de este teorema se utilizarán los siguientes resultados previos:


Lema 1 (de Schur). Sea A ∈ Mn×n con autovalores λ1 , · · · , λn reales. Entonces,
T triangular superior tal que A = QT QT . Además
existeQ ∈ Mn×n ortogonal y 
λ1 t12 · · · t1n
 .. 
 λ2 . 
T = .
 .. 
 . tn−1,n 
0 λn

Teorema 1. Sea |||·||| una norma matricial y S ∈ Mn×n una matriz invertible.
Entonces la función |||·|||S : Mn×n → R con |||A|||S = SAS −1 es una norma
matricial.
Demostración. Sea A ∈ Mn×n y |||·||| una norma matricial. Probemos primero
que ρ(A) ≤ |||A|||. Para esto consideramos un autovalor λ de A con autovector
correspondiente x. Definimos la matriz de rango 1:

.. .. ..
 
. . .
.
X = xeT = 
 
 x 0 ··· 0 .

.. .. ..
. . .

7
Observar que
.. .. .. . .. ..
   
 . . .   .. . . 
 Ax
AX =  0 ··· 0   =  λx
 0 ··· 0   = λX.
.. .. .. .. .. ..
. . . . . .

De donde |λ| |||X||| = |||λX||| = |||AX||| ≤ |||A||| |||X|||. Y dado que X ̸= 0


tenemos |λ| ≤ |||A|||. Considerando en particular el autovalor con mayor módulo
obtenemos ρ(A) ≤ |||A|||.
Sea ϵ > 0 resta probar |||A||| < ρ(A) + ϵ, por Lema de Schur1 tenemos
QAQT = T, con Q matriz ortogonal y T triangular superior cuyos elementos de
la diagonal son los autovalores de A. Sea d ∈ R, con d > 0. Definimos
 
d
 d2 0 
. 
 
Dd =  . .. .

 
n−1
 0 d 
dn
Luego,
 
dλ1 dt12 dt13 ··· dt1n

 d2 λ2 d2 t23 ··· d2 t1n 

Dd QAQT = Dd T = 
 .. .

 . 
 0 dn−1 λn−1 dn−1 tn−1,n 
dn λn

d−1 t12 d−2 t13 d−(n−1) t1n


 
λ1 ···

 λ2 d−1 t23 ··· d−(n−2) t1n 

Dd QAQT Dd−1 = Dd T Dd−1 =
 .. .

 . 
 0 λn−1 d−1 tn−1,n 
λn

Sumando en cada columna de Dd T Dd−1 el módulo de los elementos que están


arriba de la diagonal, es decir los elementos (Dd T Dd−1 )ij con i < j, tenemos
j−1
X
di−j |tij |, para cada j = 1, · · · , n. Dado que i − j < 0 podemos tomar d lo
i=1
suficientemente grande de modo que
j−1
X
máx di−j |tij | < ϵ. (1)
1≤j≤n
i=1

8
Sea B ∈ Mn×n consideramos: |||B||| = Dd QBQT Dd−1 1 = (Dd Q)B(Dd Q)−1 1
.
Por 1 tenemos |||·||| es una norma matricial. Finalmente, por teorema 1

j−1
!
X
|||A||| = Dd QAQT Dd−1 1
= máx |λj | + di−j |tij | < ρ(A) + ϵ
1≤j≤n
i=1

Corolario: IMPORTANTE Sea A ∈ Mn×n , entonces ρ(A) = ı́nf{|||A||| :


|||·||| es una norma matricial}.
Demostración. Sea A ∈ Mn×n y |||·||| una norma matricial. En el comienzo de
la demostración del teorema anterior probamos que ρ(A) es una cota inferior de

|||A|||. Pero además para todo ϵ > 0 existe una norma inducida |||·||| de modo que

ρ(A) − |||A||| < ϵ. Por lo tanto ρ(A) = ı́nf{|||A||| : |||·|||es una norma matricial}.

Teorema: IMPORTANTE Sea A ∈ Mn×n , se tiene lı́mk→∞ Ak = 0 si y


solo si ρ(A) < 1.
k→∞
Demostración. ⇒) Sea A ∈ Mn×n que satisfaceAk −→ 0 y λ alguno de sus
autovalores, con autovector x ̸= 0. Por hipótesis λk x = Ak , x → 0, por lo tanto
|λ| < 1. Ya que la desigualdad es cierta para todos los autovalores de A en
particular lo es para el que posee mayor módulo, luego ρ(A) < 1.
⇐) Sea A ∈ Mn×n tal que ρ(A) < 1, por el teorema anterior existe
k
alguna norma matricial |||·||| tal que |||A||| < 1, de donde se sigue |||A||| →
0. Además, aplicando k veces la propiedad 4. de norma matricial tenemos
k
Ak ≤ |||A||| y por lo tanto Ak → 0. Finalmente, dado que todas las
normas son equivalentes, en particular lo es la norma infinito, por lo que tenemos
Ak ∞ → 0, esto es que máx |aij | → 0. Por lo tanto, todos los elementos de
1≤i,j≤n
Ak tenderán a 0, como querı́amos.
Teorema: IMPORTANTE Sea A ∈ Mn×n y |||·||| una norma matricial,
entonces 1
ρ(A) = lı́mk→∞ Ak k .
1
Demostración. Veamos primero que ρ(A) ≤ ∥Ak ∥ k . Para esto, consideremos λ
un autovalor de A con autovector x. Luego, ∥Ak x∥ = ∥λk x∥ = |λ|k ∥x∥. De
donde obtenemos

∥Ak x∥ Ak ∥x∥
|λ|k = ≤ = Ak .
∥x∥ ∥x∥
1
De esto se sigue |λ| ≤ Ak k
. Como es cierto para todos loa autovalores
1
k
obtenemos ρ(A) ≤ A k
.

9
1
Ahora demostemos que el lı́mite de Ak k
es el radio espectral de la matriz
A.
1
Sea ϵ > 0, definimos à = ρ(A)+ϵ A, debido a que los autovalores de à son
1
ρ(A)+ϵ λi obtenemos

1 ρ(A)
ρ(Ã) = ρ(A) = < 1.
ρ(A) + ϵ ρ(A) + ϵ

Por el recı́proco del teorema anterior tenemos lı́mk→∞ Ãk = 0, es decir,


existe k0 ∈ N de modo que

1 Ak
Ãk = Ak = < 1, ∀k ≥ k0 .
(ρ(A) + ϵ)k (ρ(A) + ϵ)k
1
Esto es, Ak < (ρ(A) + ϵ)k , por lo que obtuvimos Ak k
< ρ(A) + ϵ.
1
k
Por lo tanto si k ≥ k0 se tiene ρ(A) ≤ A k
≤ ρ(A) + ϵ. Es decir
1
k k
lı́mk→∞ A = ρ(A).

Anexo: normas, productos internos y la identidad


del paralelogramo
Teorema: Toda norma inducida por un producto interno de un espacio
vectorial real satisface la identidad del paralelogramo.
Demostración. Sea (V, ⟨·, ·⟩) un espacio vectorial
preal con producto interno (p).
Por lo tanto puede definirse | · | : V → R/|x| = ⟨x, x⟩ (D) la norma inducida.
(D)
∀x, y ∈ V||x + y||2 + ||x − y||2 = ⟨x + y, x + y⟩ + ⟨x − y, x − y⟩
(p1,p2,p3)  
= ⟨x, x⟩ + 2⟨x, y⟩ + ⟨y, y⟩ + ⟨x, x⟩ − 2⟨x, y⟩ + ⟨y, y⟩
= 2⟨x, x⟩ + 2⟨y, y⟩
(D)
= 2||x||2 + 2||y||2 .

Ahora veamos que esta también es una condición suficiente para que una
norma sea inducida por un producto interno. Se necesitarán los siguientes teoremas.
Teorema: Toda forma Q-bilineal, real, simétrica y definida positiva satisface
la desigualdad de Cauchy-Schwarz.
Demostración. Sea V un espacio vectorial real. Sea ⟨·, ·⟩ : V × V → R, Q-bilineal
(1), simétrica (2) y definida positiva (3). Sean x, y ∈ V arbitrarios. Si y = 0, la
(3)
desigualdad se cumple con igualdad; se√ asume y ̸= 0. Por lo tanto ⟨y, y⟩ > 0.
ε
Sea ε ∈ R+ arbitrario. Por lo tanto ⟨y,y⟩ ∈ R+ . Por lo tanto puede definirse

⟨x,y⟩ ε ⟨x,y⟩
q ∈ Q/ q − ⟨y,y⟩ < ⟨y,y⟩ (4) una aproximación racional de ⟨y,y⟩ ∈ R.

10
(3)
⟨x, y⟩2 ≤ ⟨x, y⟩2 + ⟨x − qy, x − qy⟩⟨y, y⟩
(1,2)
≤ ⟨x, y⟩2 + (⟨x, x⟩ − 2q⟨x, y⟩ + q 2 ⟨y, y⟩)⟨y, y⟩
  2
⟨x, y⟩
= ⟨x, x⟩⟨y, y⟩ + q − ⟨y, y⟩
⟨y, y⟩
(4)
< ⟨x, x⟩⟨y, y⟩ + ε
Por lo tanto, como ε es arbitrario, ⟨x, y⟩2 ≤ ⟨x, x⟩⟨y, y⟩. Por lo tanto, como
x y y son arbitrarios, ∀x, y ∈ V⟨x, y⟩2 ≤ ⟨x, x⟩⟨y, y⟩.

Teorema: Toda función real Q-lineal continua es R-lineal.


Demostración. Sea (V, || · ||) un espacio vectorial normado real (n). Sea f : V →
(1)
R, Q-lineal (1) y continua. Por lo tanto f (0) = f (0 · 0) = 0f (0) = 0. Sean
x ∈ V/f (x) ̸= 0 y c ∈ R arbitrarios. La Q-linealidad implica aditividad, por
lo que se quiere verificar R-homogeneidad, o sea f (cx) = cf (x). El caso x = 0
es inmediato; se asume x ̸= 0. Sea ε ∈ R+ arbitrario. Como f (0) = 0 y f es
continua, puede definirse δ ∈ R+ /∀z ∈ V ||z|| < δ ⇒ |f (z)| < 2ε (2). Por lo
n o (n2)
δ
tanto puede definirse m := mı́n ||x|| , 2|fε(x)| (3). Por lo tanto m > 0. Por lo
tanto puede definirse q ∈ Q/|c − q| < m (4) una aproximación racional de c.
(n1) (4) (3) (2)
ε
||(c − q)x|| = |c − q|||x|| < m||x|| ≤ δ. Por lo tanto |f [(c − q)x]| < 2 (5).
(1)
|f (cx) − cf (x)| = |f (cx) − f (qx) + qf (x) − cf (x)|
(1)
= |f [(c − q)x] − (c − q)f (x)|
≤ |f [(c − q)x]| + |c − q||f (x)|
(4,5)
ε
< 2 + m|f (x)|
(3)
≤ ε.
Por lo tanto, como ε es arbitrario, f (cx) = cf (x). Si f (x) = 0, el argumento
δ
es similar, pero con m := ||x|| . Por lo tanto, como c y x son arbitrarios, ∀x ∈
V, c ∈ Rf (cx) = cf (x).

Ahora sı́ veamos que la identidad del paralelogramo es condición suficiente


para que una norma sea inducida por un producto interno. La demostración
presentada del siguiemte teorema es una adaptación de contenido presente en
https://en.wikipedia.org/wiki/Polarization_identity.
Teorema: Sea (V, || · ||) un espacio vectorial normado real (n). Si se cumple

∀x, y ∈ V||x + y||2 + ||x − y||2 = 2||x||2 + 2||y||2 ,

entonces existe un producto interno tal que su norma inducida es || · ||.


||x + y||2 − ||x − y||2
Demostración. Sea ⟨·, ·⟩ : V × V → R/⟨x, y⟩ = (D).
4

11
||x + x||2 − ||x − x||2
(D)
Norma inducida: ⟨x, x⟩ =
4
||2x||2 − ||0||2
=
42 2
(n1,n2) (2||x||) − 0
=
4
= ||x||2 (N ).
2 2
(D) ||x + y|| − ||x − y||
Simetrı́a: ∀x, y ∈ V⟨x, y⟩ =
4
||y + x||2 − ||(−1)(y − x)||2
=
2
4 2
(n1) ||y + x|| − (| − 1|||y − x||)
=
4
||y + x||2 − ||(y − x)||2
=
4
(D)
= ⟨y, x⟩ (p1).
(N ) (N )
Positividad definida: ∀x ∈ V[⟨x, x⟩ = ||x||2 ≥ 0∧(⟨x, x⟩ = 0 ⇔ ||x||2 = 0 ⇔
(n2)
||x|| = 0 ⇔ x = 0)]. Por lo tanto ∀x ∈ V[⟨x, x⟩ ≥ 0∧(⟨x, x⟩ = 0 ⇔ x = 0)] (p4).
Distributividad: Sean x, y, z ∈ V arbitrarios. Se tienen las siguientes cuatro
identidades del paralelogramo

||y + 2z||2 + ||y||2 = 2||y + z||2 + 2||z||2 ,

||y||2 + ||y − 2z||2 = 2||y − z||2 + 2||z||2 ,


||2x + y||2 + ||y + 2z||2 = 2||x + y + z||2 + 2||x − z||2 ,
||2x + y||2 + ||y − 2z||2 = 2||x + y − z||2 + 2||x + z||2 .
Se restan la primera con la segunda y la tercera con la cuarta

||y + 2z||2 − ||y − 2z||2 = 2||y + z||2 − 2||y − z||2 ,

||y + 2z||2 − ||y − 2z||2 = 2||x + y + z||2 + 2||x − z||2 − 2||x + y − z||2 − 2||x + z||2 .
Los lados izquierdos son iguales. Se igualan los lados derechos y se reordena

||x + y + z||2 − ||x + y − z||2 = ||x + z||2 − ||x − z||2 + ||y + z||2 − ||y − z||2 .

Se reemplaza con (D) y ya está. Como x, y y z son arbitrarios, se tiene

∀x, y, z ∈ V⟨x + y, z⟩ = ⟨x, z⟩ + ⟨y, z⟩ (p3).


* n + n
X X
Homogeneidad: Sea P (n) :≡ ∀x1 , · · · , xn , y ∈ V xi , y = ⟨xi , y⟩ el
i=1 i=1
predicado de una generalización de la propiedad
P1 distributiva. Se P procede por
1
inducción. El paso base P (1) es ∀x1 , y ∈ V i=1 ⟨xi , y⟩ = ⟨x1 , y⟩ = i=1 ⟨xi , y⟩.
El paso inductivo P (k + 1) es

12
*k+1 + * k
+
X X
∀x1 , · · · , xk+1 , y ∈ V xi , y = xi + xk+1 , y
i=1 * i=1 +
k
(p3) X
= xi , y + ⟨xk+1 , y⟩
i=1
k
P (k)X
= ⟨xi , y⟩ + ⟨xk+1 , y⟩
i=1
k+1
X
= ⟨xi , y⟩.
i=1
Por lo tanto, ∀n ∈ NP (n).*Por lo tanto
+
Xn n
X
∀x, y ∈ V, n ∈ N⟨nx, y⟩ = x, y = ⟨x, y⟩ = n⟨x, y⟩ (p2N).
i=1 i=1
||x + 0||2 − ||x − 0||2
(D)
∀x ∈ V⟨x, 0⟩ = = 0 (p0).
4
(p0,p1)
∀x, y ∈ V⟨0x, y⟩ = ⟨0, y⟩ = 0 = 0⟨x, y⟩ (p20).
(p3) (p0,p1)
∀x, y ∈ V⟨x, y⟩ + ⟨−x, y⟩ = ⟨x + (−x), y⟩ = ⟨0, y⟩ = 0. Por lo tanto
∀x, y ∈ V⟨−x, y⟩ = −⟨x, y⟩ (p2−).
(p2N) (p2−)
∀x, y ∈ V, n ∈ −N⟨nx, y⟩ = ⟨(−n)(−x), y⟩ = (−n)⟨−x, y⟩ = n⟨x, y⟩.
Esto con (p2N) y (p20), da ∀x, y ∈ V, m ∈ Z⟨mx, y⟩ = m⟨x, y⟩ (p2Z).
Dm E n Dm E (p2N) 1 D m E
∀x, y ∈ V, m ∈ Z, n ∈ N x, y = x, y = n x, y =
n n n n n
1 (p2Z) m
⟨mx, y⟩ = ⟨x, y⟩. Por lo tanto ∀x, y ∈ V, q ∈ Q⟨qx, y⟩ = q⟨x, y⟩ (p2Q).
n n
Por fin se usa el segundo teorema del anexo. Por (D), (p1), (p2Q), (p3) y
(p4), ⟨·, ·⟩ es una forma Q-bilineal, real, simétrica y definida positiva, y por lo
tanto satisface la desigualdad de Cauchy-Schwarz (cs).
Sea y ∈ V arbitrario. Por lo tanto puede definirse f : V → R/f (x) = ⟨x, y⟩
(1). Se quiere verificar que f es continua, o sea ∀x ∈ V, ε ∈ R+ ∃δ ∈ R+ ∀z ∈
V(||z − x|| < δ ⇒ |f (z) − f (x)| < ε). Sean x ∈ V y ε ∈ R+ arbitrarios. Si y = 0
(n2) (1) (p0,n2)
entonces |y| = 0 entonces ∀z ∈ V|f (z) − f (x)| = |⟨z, 0⟩ − ⟨x, 0⟩| = 0 < ε
entonces, como x y ε son arbitrarios, f es continua; se asume y ̸= 0. Por lo tanto
(n2)
ε
|y| ∈ R+ . Por lo tanto puede definirse δ := ||y|| ∈ R+ (2). Por lo tanto puede
definirse z ∈ V/||z − x|| < δ (3).
(1)
|f (z) − f (x)| = |⟨z, y⟩ − ⟨x, y⟩|
(p2−,p3)
= |⟨z − x, y⟩|
(N,cs)
≤ ||z − x||||y||
(3)
< δ||y||
(2)
= ε
Como z, x y ε son arbitrarios, f es continua.
Se usa el tercer teorema del anexo. Por (p2Q) y (p3), f es Q-lineal. Por

13
lo tanto, como es real (1) y continua, es R-lineal. Por lo tanto ∀x ∈ V, c ∈
(1) (1)
R⟨cx, y⟩ = f (cx) = cf (x) = c⟨x, y⟩. Por lo tanto, como y es arbitraria, ∀x, y ∈
V, c ∈ R⟨cx, y⟩ = c⟨x, y⟩.

14

También podría gustarte